matheraum.de
Raum für Mathematik
Offene Informations- und Nachhilfegemeinschaft

Für Schüler, Studenten, Lehrer, Mathematik-Interessierte.
Hallo Gast!einloggen | registrieren ]
Startseite · Forum · Wissen · Kurse · Mitglieder · Team · Impressum
Forenbaum
^ Forenbaum
Status Mathe
  Status Schulmathe
    Status Primarstufe
    Status Mathe Klassen 5-7
    Status Mathe Klassen 8-10
    Status Oberstufenmathe
    Status Mathe-Wettbewerbe
    Status Sonstiges
  Status Hochschulmathe
    Status Uni-Analysis
    Status Uni-Lin. Algebra
    Status Algebra+Zahlentheo.
    Status Diskrete Mathematik
    Status Fachdidaktik
    Status Finanz+Versicherung
    Status Logik+Mengenlehre
    Status Numerik
    Status Uni-Stochastik
    Status Topologie+Geometrie
    Status Uni-Sonstiges
  Status Mathe-Vorkurse
    Status Organisatorisches
    Status Schule
    Status Universität
  Status Mathe-Software
    Status Derive
    Status DynaGeo
    Status FunkyPlot
    Status GeoGebra
    Status LaTeX
    Status Maple
    Status MathCad
    Status Mathematica
    Status Matlab
    Status Maxima
    Status MuPad
    Status Taschenrechner

Gezeigt werden alle Foren bis zur Tiefe 2

Navigation
 Startseite...
 Neuerdings beta neu
 Forum...
 vorwissen...
 vorkurse...
 Werkzeuge...
 Nachhilfevermittlung beta...
 Online-Spiele beta
 Suchen
 Verein...
 Impressum
Das Projekt
Server und Internetanbindung werden durch Spenden finanziert.
Organisiert wird das Projekt von unserem Koordinatorenteam.
Hunderte Mitglieder helfen ehrenamtlich in unseren moderierten Foren.
Anbieter der Seite ist der gemeinnützige Verein "Vorhilfe.de e.V.".
Partnerseiten
Dt. Schulen im Ausland: Mathe-Seiten:Weitere Fächer:

Open Source FunktionenplotterFunkyPlot: Kostenloser und quelloffener Funktionenplotter für Linux und andere Betriebssysteme
StartseiteMatheForenUni-StochastikKlassische Stochastik
Foren für weitere Schulfächer findest Du auf www.vorhilfe.de z.B. Deutsch • Englisch • Französisch • Latein • Spanisch • Russisch • Griechisch
Forum "Uni-Stochastik" - Klassische Stochastik
Klassische Stochastik < Stochastik < Hochschule < Mathe < Vorhilfe
Ansicht: [ geschachtelt ] | ^ Forum "Uni-Stochastik"  | ^^ Alle Foren  | ^ Forenbaum  | Materialien

Klassische Stochastik: Frage (beantwortet)
Status: (Frage) beantwortet Status 
Datum: 20:51 Do 08.07.2010
Autor: matheja

Guten Abend liebe matheraumfreunde,

ich rechne grad eine altklausur durch und habe ein kleines problem mit dieser aufgabe.

Aufgabe
Seien A, B, und E Ereignisse in einem wahrscheinlichekietsraum (Omega, P)
mit P(A)=0.6, P(B)=P(E)=0.5 sowie [mm] P(B|E)=P(B|E^{c}) [/mm]

(a) Zeigen Sie, dass 0.1<= P(A [mm] \cap [/mm] B)
(b) Bestimmen sie P(B [mm] \cap [/mm] E). Begründen sie ihre antwort


Meine Idee:

zu

(b) Da [mm] gilt:P(B|E)=P(B|E^{c}) [/mm] folgt B und E sind stochastisch unabhängig
=> P(B [mm] \cap [/mm] E)=P(B)*P(E)=0.5*0.5=0.25

zu

(a) hier steh ich ein bissl auf den schlauch,
weil ich nicht weiß ob A und B stochastisch unabhängig sind, wenn ja
gilt:=> P(A [mm] \cap [/mm] B)=P(A)*P(B)=0.5*0.6=>0.1 womit die aussage gezeigt wäre, aber das ist nur die halbe wahrheit habe ich das gefühl.


Sind meine Überlegungen korrekt?
Wenn nein wo liegt der Fehler und wie kann ich das besser machen.


LG
matheja


        
Bezug
Klassische Stochastik: Antwort
Status: (Antwort) fertig Status 
Datum: 21:24 Do 08.07.2010
Autor: abakus


> Guten Abend liebe matheraumfreunde,
>  
> ich rechne grad eine altklausur durch und habe ein kleines
> problem mit dieser aufgabe.
>  
> Seien A, B, und E Ereignisse in einem
> wahrscheinlichekietsraum (Omega, P)
>  mit P(A)=0.6, P(B)=P(E)=0.5 sowie [mm]P(B|E)=P(B|E^{c})[/mm]
>  
> (a) Zeigen Sie, dass 0.1<= P(A [mm]\cap[/mm] B)
>  (b) Bestimmen sie P(B [mm]\cap[/mm] E). Begründen sie ihre
> antwort
>  
> Meine Idee:
>  
> zu
>
> (b) Da [mm]gilt:P(B|E)=P(B|E^{c})[/mm] folgt B und E sind
> stochastisch unabhängig
>  => P(B [mm]\cap[/mm] E)=P(B)*P(E)=0.5*0.5=0.25

>  
> zu
>  
> (a) hier steh ich ein bissl auf den schlauch,

Hallo,
das Ereignis A deckt 60% des Wahrscheinlichkeitsraums ab.
Wo soll B hin, ohne sich mit A zu überschneiden? Seine Abdeckung von 50% des Wahrscheinlichkeitsraums kann im günstigsten Fall die 40% belegen, die von A noch nicht abgedeckt wurden. Übrig bleiben 10%, die in A liegen MÜSSEN.
Ansonsten kommt man vielleicht mit DeMorgan weiter:
[mm] \neg(A\cap B)=\neg A\cup\neg [/mm] B.
Bekanntermaßen gilt [mm] P(X\cup Y)=P(X)+P(Y)-P(X\cap [/mm] Y), diese Beziehung könntest du auf
[mm] P(\neg [/mm] A [mm] \cup\neg [/mm] B ) anwenden...
Gruß Abakus

>  weil ich nicht weiß ob A und B stochastisch unabhängig
> sind, wenn ja
>  gilt:=> P(A [mm]\cap[/mm] B)=P(A)*P(B)=0.5*0.6=>0.1 womit die

> aussage gezeigt wäre, aber das ist nur die halbe wahrheit
> habe ich das gefühl.
>  
>
> Sind meine Überlegungen korrekt?
>  Wenn nein wo liegt der Fehler und wie kann ich das besser
> machen.
>  
>
> LG
>  matheja
>  


Bezug
                
Bezug
Klassische Stochastik: herzlichen dank
Status: (Mitteilung) Reaktion unnötig Status 
Datum: 22:26 Do 08.07.2010
Autor: matheja

danke für für die tolle anschauliche erklärung


mfg

matheja

Bezug
                
Bezug
Klassische Stochastik: Frage (beantwortet)
Status: (Frage) beantwortet Status 
Datum: 23:51 Do 08.07.2010
Autor: kegel53

Sorry, dass ich mich einmische, aber bin grad auf die Frage gestoßen und wunder mich wie hierbei wie DeMorgan helfen kann?!

Es ist doch [mm] P[A\cap{B}]=P[A]+P[B]-P[A\cup{B}] [/mm] oder mit deinem Vorschlag [mm] P[A\cap{B}]=1-P[A^c\cup{B^c}]=1-(P[A^c]+P[B^c]-P[A^c\cap{B^c}]). [/mm]

D.h. in beiden Varianten kann ich nicht alle W'keiten berechnen, in der ersten macht [mm] P[A\cup{B}] [/mm] Probleme und in der zweiten [mm] P[A^c\cap{B^c}], [/mm] was auf das gleiche rausläuft.
Oder seh ich im Moment schlichtweg was falsch und die Berechnung geht womöglich doch ganz einfach von der Hand??

Bezug
                        
Bezug
Klassische Stochastik: Antwort
Status: (Antwort) fertig Status 
Datum: 02:13 Fr 09.07.2010
Autor: Marcel

Hallo,

> Sorry, dass ich mich einmische, aber bin grad auf die Frage
> gestoßen und wunder mich wie hierbei wie DeMorgan helfen
> kann?!
>  
> Es ist doch [mm]P[A\cap{B}]=P[A]+P[B]-P[A\cup{B}][/mm] oder mit
> deinem Vorschlag
> [mm]P[A\cap{B}]=1-P[A^c\cup{B^c}]=1-(P[A^c]+P[B^c]-P[A^c\cap{B^c}]).[/mm]
>  
> D.h. in beiden Varianten kann ich nicht alle W'keiten
> berechnen, in der ersten macht [mm]P[A\cup{B}][/mm] Probleme und in
> der zweiten [mm]P[A^c\cap{B^c}],[/mm] was auf das gleiche
> rausläuft.
>  Oder seh ich im Moment schlichtweg was falsch und die
> Berechnung geht womöglich doch ganz einfach von der Hand??

es ist eigentlich sehr einfach, und ihr habt doch schon alles gesagt (de Morgan braucht man nicht, aber "Rechenregeln" für Wahrscheinlichkeitsmaße bzw. (endliche) Maße):

Aus $P(A [mm] \cup [/mm] B)+P(A [mm] \cap [/mm] B)=P(A)+P(B)$ folgt

$$P(A [mm] \cap [/mm] B)=P(A)+P(B)-P(A [mm] \cup [/mm] B)=1,1-P(A [mm] \cup B)\,.$$ [/mm]

Wegen $P(A [mm] \cup [/mm] B) [mm] \le P(\Omega)=1$ [/mm] (da $A [mm] \cup [/mm] B [mm] \subseteq \Omega$) [/mm] folgt
$$P(A [mm] \cap [/mm] B)=1,1-P(A [mm] \cup [/mm] B) [mm] \ge 1,1-P(\Omega)=1,1-1=0,1\,.$$ [/mm]

Beste Grüße,
Marcel

Bezug
                                
Bezug
Klassische Stochastik: Mitteilung
Status: (Mitteilung) Reaktion unnötig Status 
Datum: 02:59 Fr 09.07.2010
Autor: kegel53

Vielen Dank Marcel!!
Das hat mich jetzt doch arg beschäftigt, da konnt ich ohne Lösung nicht ins Bett :).

Bezug
                
Bezug
Klassische Stochastik: Mit de Morgan
Status: (Mitteilung) Reaktion unnötig Status 
Datum: 02:31 Fr 09.07.2010
Autor: Marcel

Hallo,

> > Guten Abend liebe matheraumfreunde,
>  >  
> > ich rechne grad eine altklausur durch und habe ein kleines
> > problem mit dieser aufgabe.
>  >  
> > Seien A, B, und E Ereignisse in einem
> > wahrscheinlichekietsraum (Omega, P)
>  >  mit P(A)=0.6, P(B)=P(E)=0.5 sowie [mm]P(B|E)=P(B|E^{c})[/mm]
>  >  
> > (a) Zeigen Sie, dass 0.1<= P(A [mm]\cap[/mm] B)
>  >  (b) Bestimmen sie P(B [mm]\cap[/mm] E). Begründen sie ihre
> > antwort
>  >  
> > Meine Idee:
>  >  
> > zu
> >
> > (b) Da [mm]gilt:P(B|E)=P(B|E^{c})[/mm] folgt B und E sind
> > stochastisch unabhängig
>  >  => P(B [mm]\cap[/mm] E)=P(B)*P(E)=0.5*0.5=0.25

>  >  
> > zu
>  >  
> > (a) hier steh ich ein bissl auf den schlauch,
>  Hallo,
>  das Ereignis A deckt 60% des Wahrscheinlichkeitsraums ab.
>  Wo soll B hin, ohne sich mit A zu überschneiden? Seine
> Abdeckung von 50% des Wahrscheinlichkeitsraums kann im
> günstigsten Fall die 40% belegen, die von A noch nicht
> abgedeckt wurden. Übrig bleiben 10%, die in A liegen
> MÜSSEN.
>  Ansonsten kommt man vielleicht mit DeMorgan weiter:
>  [mm]\neg(A\cap B)=\neg A\cup\neg[/mm] B.
>  Bekanntermaßen gilt [mm]P(X\cup Y)=P(X)+P(Y)-P(X\cap[/mm] Y),
> diese Beziehung könntest du auf
> [mm]P(\neg[/mm] A [mm]\cup\neg[/mm] B ) anwenden...

naja, das ganze folgt sehr schnell aus $A [mm] \cup [/mm] B [mm] \subseteq \Omega\,.$ [/mm] Man kann aber vll. durchaus auch mit de Morgan argumentieren:
$$P((A [mm] \cap B)^c)=P(A^c)+P(B^c)-P(A^c \cap B^c)=0,4+0,5-P(A^c \cap B^c)\,.$$ [/mm]

Da [mm] $P(A^c \cap B^c) \ge [/mm] 0$ ist daher $P((A [mm] \cap B)^c)\le [/mm] 0,9,$ woraus
$$P(A [mm] \cap [/mm] B)=1-P((A [mm] \cap B)^c) \ge [/mm] 1-0,9=0,1$$
folgt.

Beste Grüße,
Marcel

Bezug
        
Bezug
Klassische Stochastik: Antwort
Status: (Antwort) fertig Status 
Datum: 02:23 Fr 09.07.2010
Autor: Marcel

Hallo,

> Guten Abend liebe matheraumfreunde,
>  
> ich rechne grad eine altklausur durch und habe ein kleines
> problem mit dieser aufgabe.
>  
> Seien A, B, und E Ereignisse in einem
> wahrscheinlichekietsraum (Omega, P)
>  mit P(A)=0.6, P(B)=P(E)=0.5 sowie [mm]P(B|E)=P(B|E^{c})[/mm]
>  
> (a) Zeigen Sie, dass 0.1<= P(A [mm]\cap[/mm] B)
>  (b) Bestimmen sie P(B [mm]\cap[/mm] E). Begründen sie ihre
> antwort
>  
> Meine Idee:
>  
> zu
>
> (b) Da [mm]gilt:P(B|E)=P(B|E^{c})[/mm] folgt B und E sind
> stochastisch unabhängig

stimmt, siehe etwa []hier.

>  => P(B [mm]\cap[/mm] E)=P(B)*P(E)=0.5*0.5=0.25

Ich sehe da jedenfalls keinen (Denk- oder Rechen-)fehler.
  

> zu
>  
> (a) hier steh ich ein bissl auf den schlauch,
>  weil ich nicht weiß ob A und B stochastisch unabhängig
> sind, wenn ja
>  gilt:=> P(A [mm]\cap[/mm] B)=P(A)*P(B)=0.5*0.6=>0.1 womit die

> aussage gezeigt wäre, aber das ist nur die halbe wahrheit
> habe ich das gefühl.

Sie müssen nicht unabhängig sein. Benutze
$$P(A [mm] \cap [/mm] B)=1,1-P(A [mm] \cup B)\;\;\text{(Frage an Dich: Wie kommt die 1,1 zustande?)}$$ [/mm]
und schätze $P(A [mm] \cup [/mm] B)$ mit $A [mm] \cup [/mm] B [mm] \subseteq \Omega$ [/mm] ab. Wenn Du nicht weiterkommst, ich habe es hier (in Deinem Thread) auch genauer ausgeführt. Du wirst Dich vll. wundern, wie banal es für Dich ist, wenn Du meine Lösung mal gesehen hat (ist ja meistens so: Wenn man die Lösung kennt, wundert man sich, dass man da nicht alleine drauf gekommen ist ^^).

Beste Grüße,
Marcel

Bezug
Ansicht: [ geschachtelt ] | ^ Forum "Uni-Stochastik"  | ^^ Alle Foren  | ^ Forenbaum  | Materialien


^ Seitenanfang ^
www.matheraum.de
[ Startseite | Forum | Wissen | Kurse | Mitglieder | Team | Impressum ]